- Sun Jan 20, 2013 12:00 am
#26042
Complete Question Explanation
(The complete setup for this game can be found here: lsat/viewtopic.php?f=107&t=7219)
The correct answer choice is (A)
If each zone contains an I subzone, then no zone can include an H subzone in compliance with the last rule. This helps eliminate answer choice (B). And, since Z1 does not contain any R subzones, it follows that Z1 is limited to exactly one subzone—eliminating answer choice (C):
Answer choice (D) is incorrect, because we do not have enough variables to designate three subzones each in Z2 and Z3. This would require a total of four more variables, which is impossible given the requirement that no more than 3 subzones are designated for each use.
Answer choice (E) is also incorrect, because we cannot designate four subzones in Z3. Having maxed out the number of I subzones available (3), we can only rely on R subzones to supplement either Z2 or Z3. However, the last rule forbids us from placing 3 R subzones in any zone containing I subzones (I 3R), which means that the greatest number of subzones in either Z2 or Z3 is three, not four.
(The complete setup for this game can be found here: lsat/viewtopic.php?f=107&t=7219)
The correct answer choice is (A)
If each zone contains an I subzone, then no zone can include an H subzone in compliance with the last rule. This helps eliminate answer choice (B). And, since Z1 does not contain any R subzones, it follows that Z1 is limited to exactly one subzone—eliminating answer choice (C):
Answer choice (D) is incorrect, because we do not have enough variables to designate three subzones each in Z2 and Z3. This would require a total of four more variables, which is impossible given the requirement that no more than 3 subzones are designated for each use.
Answer choice (E) is also incorrect, because we cannot designate four subzones in Z3. Having maxed out the number of I subzones available (3), we can only rely on R subzones to supplement either Z2 or Z3. However, the last rule forbids us from placing 3 R subzones in any zone containing I subzones (I 3R), which means that the greatest number of subzones in either Z2 or Z3 is three, not four.
You do not have the required permissions to view the files attached to this post.